summaryrefslogtreecommitdiff
path: root/Master/texmf-dist/doc/latex/bhcexam/test2.tex
diff options
context:
space:
mode:
authorKarl Berry <karl@freefriends.org>2014-06-23 23:26:54 +0000
committerKarl Berry <karl@freefriends.org>2014-06-23 23:26:54 +0000
commiteebd4ae245ee096e3f02f10db88408cb906aa7ec (patch)
treecd9961d84df8bb8f46ded64c200e8a1920931ac0 /Master/texmf-dist/doc/latex/bhcexam/test2.tex
parent54d546a4891626936992e9cdf0b1cdf4e6cc0aeb (diff)
bhcexam (23jun14)
git-svn-id: svn://tug.org/texlive/trunk@34360 c570f23f-e606-0410-a88d-b1316a301751
Diffstat (limited to 'Master/texmf-dist/doc/latex/bhcexam/test2.tex')
-rw-r--r--Master/texmf-dist/doc/latex/bhcexam/test2.tex213
1 files changed, 0 insertions, 213 deletions
diff --git a/Master/texmf-dist/doc/latex/bhcexam/test2.tex b/Master/texmf-dist/doc/latex/bhcexam/test2.tex
deleted file mode 100644
index ac2c041c24c..00000000000
--- a/Master/texmf-dist/doc/latex/bhcexam/test2.tex
+++ /dev/null
@@ -1,213 +0,0 @@
-\documentclass[16kpaper]{BHCexam}
-\begin{document}
-
-\maketitle
-\mininotice
-
-\begin{questions}
-\tiankong
-\question 已知~$\bm{a}=(k,-9)$、$\bm{b}=(-1,k)$, $\bm{a}$~与~$\bm{b}$~为平行向量,
- 则~$k=$\sixb.
-\begin{solution}
-$\pm3$
-\end{solution}
-\question 若函数~$f(x)=x^{6m^2-5m-4}\,(m\in\mathbb{Z})$~的图像关于~$y$~轴对称,
- 且~$f(2)<f(6)$, 则~$f(x)$~的解析式为\tenb.
-\begin{solution}
-$f(x)=x^{-4}$
-\end{solution}
-
-\question 若~$f(x+1)=x^2\,(x\leq0)$, 则~$f^{-1}(1)=$\sixb.
-\begin{solution}
-0
-\end{solution}
-
-\question 在~$b\g$~糖水中含糖~$a\g$\,($b>a>0$), 若再添加~$m\g$~糖~($m>0$),
- 则糖水就变甜了.试根据这个事实, 提炼一个不等式\tenb.
-\begin{solution}
-$\dfrac{a+m}{b+n}>\dfrac{a}{b}$
-\end{solution}
-
-\question 已知~$f(x)=1-\rc_8^1x+\rc_8^2x^2-\rc_8^3x^3+\cdots+\rc_8^8x^8$,
- 则~$f\big(\dfrac{1}{2}+\dfrac{\sqrt{3}}{2}\ri\big)$~的值是\sixb\twob{}.
-\begin{solution}
-$-\dfrac{1}{2}-\dfrac{\sqrt{3}}{2}\ri$
-\end{solution}
-
-\question 自然数~$1,2,3,\ldots,10$~的方差记为~$\sigma^2$,
- 其中的偶数~$2,4,6,8,10$~的方差记为~$\sigma_1^2$,
- 则~$\sigma^2$~与~$\sigma_1^2$~的大小关系为~$\sigma^2$\sixb$\sigma_1^2$.
-\begin{solution}
-$>$
-\end{solution}
-
-\question 若~$\theta$~为三角形的一个内角, 且~$\sin\theta+\cos\theta=\dfrac{2}{3}$,
- 则方程~$x^2\csc\theta-y^2\sec\theta=1$~表示的曲线的焦点坐标是\sixb{}.
-\begin{solution}
-$\big(\pm\dfrac{\sqrt{6}}{3},0\big)$
-\end{solution}
-
-\question 高为~$h$~的棱锥被平行于棱锥底面的截得棱台侧面积是
- 原棱锥的侧面积的~$\dfrac{5}{9}$,
- 则截得的棱台的体积与原棱锥的体积之比是\sixb.
-\begin{solution}
-$19:27$
-\end{solution}
-
-\question 以椭圆~$\dfrac{x^2}{169}+\dfrac{y^2}{144}=1$~的右焦点为圆心,
- 且与双曲线~$\dfrac{x^2}{9}-\dfrac{y^2}{16}=1$~的渐近线相切的圆方程是\tenb.
-\begin{solution}
-$(x-5)^2+y^2=16$
-\end{solution}
-
-\question 若~$\sqrt{\,\sin x}$~是有理数且~$x$~不是~$\dfrac{\pi}{6}$~的整数倍,
- 则~$x$~可能取的值是\tenb.(只要求写出一个)
-\begin{solution}
-$\arcsin\dfrac{1}{4}$ 等
-\end{solution}
-
-\question 马路上有编号~1~到~10~的~10~盏路灯, 为节约用电又不影响照明,
- 可以关掉其中的~3~盏, 但又不能同时关掉相邻的两盏, 也不能关掉两端的路灯,
- 满足条件的关灯方法有\sixb{}种.
-\begin{solution}
-20
-\end{solution}
-\question 以椭圆~$\dfrac{x^2}{169}+\dfrac{y^2}{144}=1$~的右焦点为圆心,
- 且与双曲线~$\dfrac{x^2}{9}-\dfrac{y^2}{16}=1$~的渐近线相切的圆方程是\tenb.
-\begin{solution}
-$(x-5)^2+y^2=16$
-\end{solution}
-
-\newpage
-
-\question 若~$\sqrt{\,\sin x}$~是有理数且~$x$~不是~$\dfrac{\pi}{6}$~的整数倍,
- 则~$x$~可能取的值是\tenb.(只要求写出一个)
-\begin{solution}
-$\arcsin\dfrac{1}{4}$ 等
-\end{solution}
-
-\question 马路上有编号~1~到~10~的~10~盏路灯, 为节约用电又不影响照明,
- 可以关掉其中的~3~盏, 但又不能同时关掉相邻的两盏, 也不能关掉两端的路灯,
- 满足条件的关灯方法有\sixb{}种.
-\begin{solution}
-20
-\end{solution}
-\xuanze
-\question 已知集合~$A=\left\{\,x\mid \abs{x-1}<3\,\right\}$,
-集合~$B=\{\,y\mid y=x^2+2x+1,x\in\mathbb{R}\,\}$, 则~$A\cap
-\complement_U B$~为
-\begin{choices}
-\choice $[\,0,4)$
-\choice $(-\infty,-2\,]\cup[4,+\infty)$
-\choice $(-2,0)$
-\choice $(0,4)$
-\end{choices}
-\begin{solution}
-C
-\end{solution}
-
-\question 若~$a$、$b$~是直线, $\alpha$、$\beta$~是平面,
-则以下命题中真命题是
-\begin{choices}
-\choice 若~$a$、$b$~异面, $a\subset\alpha$,
-$b\subset\beta$, 且~$a\perp b$, 则~$\alpha\perp\beta$
-\choice 若~$a\pingxing b$, $a\subset\alpha$, $b\subset\beta$,
-则~$\alpha\pingxing\beta$
-\choice 若~$a\pingxing \alpha$,
-$b\subset\beta$, 则~$a$、$b$ 异面
-\choice 若~$a\perp b$, $a\perp\alpha$,
-$b\perp\beta$, 则~$\alpha\perp\beta$
-\end{choices}
-\begin{solution}
-D
-\end{solution}
-
-\question 已知集合~$A=\left\{\,x\mid \abs{x-1}<3\,\right\}$,
-集合~$B=\{\,y\mid y=x^2+2x+1,x\in\mathbb{R}\,\}$, 则~$A\cap
-\complement_U B$~为
-\begin{choices}
-\choice $[\,0,4)$
-\choice $(-\infty,-2\,]\cup[4,+\infty)$
-\choice $(-2,0)$
-\choice $(0,4)$
-\end{choices}
-\begin{solution}
-C
-\end{solution}
-
-\question 若~$a$、$b$~是直线, $\alpha$、$\beta$~是平面,
-则以下命题中真命题是
-\begin{choices}
-\choice 若~$a$、$b$~异面, $a\subset\alpha$,
-$b\subset\beta$, 且~$a\perp b$, 则~$\alpha\perp\beta$
-\choice 若~$a\pingxing b$, $a\subset\alpha$, $b\subset\beta$,
-则~$\alpha\pingxing\beta$
-\choice 若~$a\pingxing \alpha$,
-$b\subset\beta$, 则~$a$、$b$ 异面
-\choice 若~$a\perp b$, $a\perp\alpha$,
-$b\perp\beta$, 则~$\alpha\perp\beta$
-\end{choices}
-\begin{solution}
-D
-\end{solution}
-\newpage
-\jianda
-\question 已知复数~$z$ 满足:$\abs{z}-z^*=\dfrac{10}{1-w\ri}$(其中~$z^*$
-是~$z$ 的共轭复数).
-\begin{parts}
-\part[7] 求复数~$z$;
-\part[7] 若复数~$w=\cos\theta+\ri\sin\theta\,(\theta\in\mathbb{R})$, 求~$\abs{z-2}$ 的取值范围.
-\end{parts}
-
-\begin{solution}
-\begin{parts}
-\part $z=3+4\ri$
-\part $\abs{z-w}\in[4,6]$
-\end{parts}
-\end{solution}
-
-\question[14] 函数~$f(x)=4\sin\dfrac{\pi}{12}x\cdot\sin
- \left(\dfrac{\pi}{2}+\dfrac{\pi}{12}x\right),x\in[a,a+1]$,
- 其中常数~$a\in[0,5]$, 求函数~$f(x)$ 的最大值~$g(a)$.
-
-\begin{solution}
-略
-\end{solution}
-
-\newpage
-
-\question[16] 函数~$f(x)=4\sin\dfrac{\pi}{12}x\cdot\sin
- \left(\dfrac{\pi}{2}+\dfrac{\pi}{12}x\right),x\in[a,a+1]$,
- 其中常数~$a\in[0,5]$, 求函数~$f(x)$ 的最大值~$g(a)$.
-
-\begin{solution}
-略
-\end{solution}
-
-\newpage
-\question 已知复数~$z$ 满足:$\abs{z}-z^*=\dfrac{10}{1-w\ri}$(其中~$z^*$
-是~$z$ 的共轭复数).
-\begin{parts}
-\part[8] 求复数~$z$;
-\part[8] 若复数~$w=\cos\theta+\ri\sin\theta\,(\theta\in\mathbb{R})$, 求~$\abs{z-2}$ 的取值范围.
-\end{parts}
-
-\begin{solution}
-\begin{parts}
-\part $z=3+4\ri$
-\part $\abs{z-w}\in[4,6]$
-\end{parts}
-\end{solution}
-
-\newpage
-
-\question[18] 函数~$f(x)=4\sin\dfrac{\pi}{12}x\cdot\sin
- \left(\dfrac{\pi}{2}+\dfrac{\pi}{12}x\right),x\in[a,a+1]$,
- 其中常数~$a\in[0,5]$, 求函数~$f(x)$ 的最大值~$g(a)$.
-
-\begin{solution}
-略
-\end{solution}
-
-\end{questions}
-\end{document}